Difference between revisions of "2013 AMC 12B Problems/Problem 19"
(→Problem) |
Mathkiddus (talk | contribs) (→Solution 3) |
||
(68 intermediate revisions by 25 users not shown) | |||
Line 5: | Line 5: | ||
In triangle <math>ABC</math>, <math>AB=13</math>, <math>BC=14</math>, and <math>CA=15</math>. Distinct points <math>D</math>, <math>E</math>, and <math>F</math> lie on segments <math>\overline{BC}</math>, <math>\overline{CA}</math>, and <math>\overline{DE}</math>, respectively, such that <math>\overline{AD}\perp\overline{BC}</math>, <math>\overline{DE}\perp\overline{AC}</math>, and <math>\overline{AF}\perp\overline{BF}</math>. The length of segment <math>\overline{DF}</math> can be written as <math>\frac{m}{n}</math>, where <math>m</math> and <math>n</math> are relatively prime positive integers. What is <math>m+n</math>? | In triangle <math>ABC</math>, <math>AB=13</math>, <math>BC=14</math>, and <math>CA=15</math>. Distinct points <math>D</math>, <math>E</math>, and <math>F</math> lie on segments <math>\overline{BC}</math>, <math>\overline{CA}</math>, and <math>\overline{DE}</math>, respectively, such that <math>\overline{AD}\perp\overline{BC}</math>, <math>\overline{DE}\perp\overline{AC}</math>, and <math>\overline{AF}\perp\overline{BF}</math>. The length of segment <math>\overline{DF}</math> can be written as <math>\frac{m}{n}</math>, where <math>m</math> and <math>n</math> are relatively prime positive integers. What is <math>m+n</math>? | ||
− | <math> \textbf{(A)}\ 18\qquad\textbf{(B)}\ 21\qquad\textbf{(C)}\ 24\qquad\textbf{(D) | + | <math> \textbf{(A)}\ 18\qquad\textbf{(B)}\ 21\qquad\textbf{(C)}\ 24\qquad\textbf{(D)}\ 27\qquad\textbf{(E)}\ 30</math> |
[[Category: Introductory Geometry Problems]] | [[Category: Introductory Geometry Problems]] | ||
+ | |||
+ | ==Diagram== | ||
+ | <asy> | ||
+ | size(200); | ||
+ | defaultpen(linewidth(0.4)+fontsize(10)); | ||
+ | pen s = linewidth(0.8)+fontsize(8); | ||
+ | |||
+ | pair A,B,C,D,E0,F,G; | ||
+ | A = origin; | ||
+ | C = (15,0); | ||
+ | B = IP(CR(A,13),CR(C,14)); | ||
+ | D = foot(A,C,B); | ||
+ | E0 = foot(D,A,C); | ||
+ | F = OP(CR((A+B)/2,length(B-A)/2), D--E0); | ||
+ | draw(A--C--B--A, black+0.8); | ||
+ | draw(B--F--A--D--E0); | ||
+ | dot("$A$",A,W); | ||
+ | dot("$B$",B,N); | ||
+ | dot("$C$",C,E); | ||
+ | dot("$D$",D,NE); | ||
+ | dot("$E$",E0,S); | ||
+ | dot("$F$",F,E); | ||
+ | draw(rightanglemark(B,D,A,15)); | ||
+ | draw(rightanglemark(B,F,A,15)); | ||
+ | draw(rightanglemark(D,E0,A,15)); | ||
+ | label("$5$",D--B,NE); | ||
+ | label("$9$",D--C,NE); | ||
+ | label(Label("$13$",Rotate(B-A)), B--A); | ||
+ | </asy> | ||
==Solution 1== | ==Solution 1== | ||
− | Since <math>\angle{AFB}=\angle{ADB}=90</math>, quadrilateral <math>ABDF</math> is cyclic. It follows that <math>\angle{ADE}=\angle{ABF}</math>. In addition, | + | Since <math>\angle{AFB}=\angle{ADB}=90^{\circ}</math>, quadrilateral <math>ABDF</math> is cyclic. It follows that <math>\angle{ADE}=\angle{ABF}</math>, so <math>\triangle ABF \sim \triangle ADE</math> are similar. In addition, <math>\triangle ADE \sim \triangle ACD</math>. We can easily find <math>AD=12</math>, <math>BD = 5</math>, and <math>DC=9</math> using Pythagorean triples. |
+ | |||
+ | So, the ratio of the longer leg to the hypotenuse of all three similar triangles is <math>\tfrac{12}{15} = \tfrac{4}{5}</math>, and the ratio of the shorter leg to the hypotenuse is <math>\tfrac{9}{15} = \tfrac{3}{5}</math>. It follows that <math>AF=(\tfrac{4}{5}\cdot 13), BF=(\tfrac{3}{5}\cdot 13)</math>. | ||
+ | |||
+ | Let <math>x=DF</math>. By [[Ptolemy's Theorem|Ptolemy's Theorem]], we have <cmath>13x+\left(5\cdot 13\cdot \frac{4}{5}\right)= 12\cdot 13\cdot \frac{3}{5} \qquad \Leftrightarrow \qquad 13x+52=93.6.</cmath> Dividing by <math>13</math> we get <math>x+4=7.2\implies x=\frac{16}{5}</math> so our answer is <math>\boxed{\textbf{(B) }21}</math>. | ||
+ | |||
+ | ~Edits by BakedPotato66 | ||
==Solution 2== | ==Solution 2== | ||
− | + | From solution 1, we know that <math>AD = 12</math> and <math>DC = 9</math>. Since <math>\triangle ADC \sim \triangle DEC</math>, we can figure out that <math>DE = \frac{36}{5}</math>. We also know what <math>AC</math> is so we can figure what <math>AE</math> is: <math>AE = 15 - \frac{27}{5} = \frac{48}{5}</math> . Quadrilateral <math>ABDF</math> is cyclic, implying that <math>\angle{B} + \angle{DFA}</math> = 180°. Therefore, <math>\angle{B} = 180 - \angle{DFA} = \angle{EFA}</math>, and triangles <math>\triangle AEF \sim \triangle ADB</math>. Solving the resulting proportion gives <math>EF = 4</math>. Therefore, <math>DF = ED - EF = \frac{16}{5}</math>. <math>m + n = 16 + 5 = 21</math> and our answer is <math>\boxed{\textbf{(B) } 21}</math>. | |
+ | |||
+ | <math>\triangle ADC \sim \triangle DEC</math> because of <math>AA \sim</math> . <math>\angle{ADC} = \angle{DEC} = 90°</math>. Lets say <math>\angle{ADE} = x</math>. So <math>\angle{EDC} = 90 - x</math> and <math>\angle{DEC} = 180 - 90 - (90 - x) = x</math> so <math>\angle{ADE} = \angle{DEC}</math> | ||
+ | |||
+ | ~[https://artofproblemsolving.com/wiki/index.php/User:South South] | ||
+ | |||
+ | ==Solution 3== | ||
+ | Let <math>\angle{ADF} = \theta,</math> and <math>DF = x.</math> Noting that <math>AD</math> is the altitude from <math>A</math> to <math>BC,</math> <math>AD=12,</math> <math>BD = 5,</math> and <math>DC = 9.</math> Notice that since <math>DE</math> is perpendicular to <math>AC</math>, <math>\triangle{ADC}</math> is similar to <math>\triangle{AEC}.</math> Therefore since <math>\triangle{ADC}</math> is a <math>3</math>-<math>4</math>-<math>5</math> triangle, <math>\cos{\theta}=\frac{3}{5}.</math> | ||
+ | Applying Law of Cosines on <math>\triangle{ADF}</math> we have,<cmath>AF^2 = 144+x^2 - 2\cdot 12 \cdot x \cdot \cos{\theta} = 144+x^2 - \frac{72x}{5}.</cmath> Next notice that <math>\cos{\angle{BDF}} = \cos{(90 + \theta)} = -\sin{\theta} = \frac{-4}{5}.</math> Now we apply Law of Cosines on <math>\triangle{BDF},</math> | ||
+ | <cmath>BF^2 = 25 + x^2 - 2 \cdot 5 \cdot x \cdot \cos{\angle{BDF}} = 25 + x^2 + 8x.</cmath> | ||
+ | Finally using Pythagorean Theorem on <math>\triangle{ABF}</math> we have, <cmath>AB^2 = AF^2 + BF^2.</cmath> <cmath>13^2 = 144+x^2 - \frac{72x}{5} + 25 + x^2 + 8x.</cmath> <cmath>0 = 2x^2 - \frac{32x}{5}.</cmath> Solving for the nonnegative value for <math>x</math> we find <math>x = DF = \frac{16}{5} \implies \boxed{21}.</math> | ||
+ | |||
+ | ~[[User:Mathkiddus|mathkiddus]] | ||
+ | |||
+ | ==Solution 4== | ||
+ | If we draw a diagram as given, but then add point <math>G</math> on <math>\overline{BC}</math> such that <math>\overline{FG}\perp\overline{BC}</math> in order to use the Pythagorean theorem, we end up with similar triangles <math>\triangle{DFG}</math> and <math>\triangle{DCE}</math>. Thus, <math>FG=\tfrac35x</math> and <math>DG=\tfrac45x</math>, where <math>x</math> is the length of <math>\overline{DF}</math>. Using the Pythagorean theorem, we now get <cmath>BF = \sqrt{\left(\frac45x+ 5\right)^2 + \left(\frac35x\right)^2}</cmath> and <math>AF</math> can be found out noting that <math>AE</math> is just <math>\tfrac{48}5</math> through base times height (since <math>12\cdot 9 = 15 \cdot \tfrac{36}5</math>, similar triangles gives <math>AE = \tfrac{48}5</math>), and that <math>EF</math> is just <math>\tfrac{36}5 - x</math>. From there, <cmath>AF = \sqrt{\left(\frac{36}5 - x\right)^2 + \left(\frac{48}5\right)^2}.</cmath> Now, <math>BF^2 + AF^2 = 169</math>, and squaring and adding both sides and subtracting a 169 from both sides gives <math>2x^2 - \tfrac{32}5x = 0</math>, so <math>x = \tfrac{16}5</math>. Thus, the answer is <math>\boxed{\textbf{(B)}}</math>. | ||
+ | |||
+ | ==Solution 5 (Power of a Point)== | ||
+ | First, we find <math>BD = 5</math>, <math>DC = 9</math>, and <math>AD = 12</math> via the Pythagorean Theorem or by using similar triangles. Next, because <math>DE</math> is an altitude of triangle <math>ADC</math>, <math>DE = \frac{AD\cdot DC}{AC} = \frac{36}{5}</math>. Using that, we can use the Pythagorean Theorem and similar triangles to find <math>EC = \frac{27}{5}</math> and <math>AE = \frac{48}{5}</math>. | ||
+ | |||
+ | Points <math>A</math>, <math>B</math>, <math>D</math>, and <math>F</math> all lie on a circle whose diameter is <math>AB</math>. Let the point where the circle intersects <math>AC</math> be <math>G</math>. Using power of a point, we can write the following equation to solve for <math>AG</math>: <cmath>DC\cdot BC = CG\cdot AC</cmath> <cmath>9\cdot 14 = CG\cdot 15</cmath> <cmath>CG = 126/15</cmath> Using that, we can find <math>AG = \frac{99}{15}</math>, and using <math>AG</math>, we can find that <math>GE = 3</math>. | ||
+ | |||
+ | We can use power of a point again to solve for <math>DF</math>: <cmath>FE\cdot DE = GE\cdot AE</cmath> <cmath>(\frac{36}{5} - DF)\cdot \frac{36}{5} = 3 \cdot \frac{48}{5}</cmath> <cmath>\frac{36}{5} - DF = 4</cmath> <cmath>DF = \frac{16}{5} = \frac{m}{n}</cmath> Thus, <math>m+n = 16+5 = 21</math> <math>\boxed{\textbf{(B)}}</math>. | ||
+ | |||
+ | ==Solution 6 (Coord Bash)== | ||
+ | If we draw the diagram like above, but make <math>BC</math> the base, we can set <math>B(0,0), C(14,0)</math> and <math>A(5,12)</math>, as <math>AD</math> can quickly be verified to be <math>12</math> by Pythagorean triples or similar triangles. Construct <math>X</math> on <math>BC</math> such that <math>EX \perp BC</math>. This implies <math>\triangle ADC \sim \triangle EXC</math> as <math>AD \parallel EX</math>, and <math> \angle ACD = \angle ECX </math>. Also construct <math>FY</math> such that <math>FY \perp BC</math>. | ||
+ | |||
+ | |||
+ | Line <math>\overline {AC}</math> has a slope of <math>-\frac{4}{3}</math> by slope formula. Since <math>D = (5,0)</math> and <math>DE \perp AC,</math>, the equation of <math> DE = \frac{3}{4}x - \frac{15}{4}</math>. | ||
+ | |||
+ | Furthermore, <math>F</math> can now be expressed as <math>(x,\frac{3}{4}x - \frac{15}{4}).</math> Since we know <math>AF \perp BF,</math> we can solve for <math>x</math> with the perpendicular slope formula like so: <cmath>\frac{\frac{3}{4}x - \frac{63}{4}}{x - 5} \cdot \frac{\frac{3}{4}x - \frac {15}{4}}{x} = -1</cmath> <cmath>\frac{x-21}{x-5} \cdot \frac{x-5}{x} = -\frac{16}{9}</cmath> <cmath>\frac{x-21}{x} = -\frac{16}{9}</cmath> <cmath> 9x = 189 -16x </cmath> <cmath>x = \frac{189}{25}</cmath> | ||
+ | |||
+ | Plugging <math>\frac{189}{25}</math> into <math>F</math>, we get <math>F(\frac{189}{25},\frac{48}{25}).</math> Since <math>FY \perp DY</math>, we get that <math>\triangle FYD</math> has side lengths of <math>FY = \frac{48}{25}</math> | ||
+ | |||
+ | and <math>DY = BY - BD = BY - 5 = \frac{64}{25}</math>. | ||
+ | |||
+ | Clearly, <math>\triangle FYD</math> is a <math>3 - 4 - 5</math> pythagorean triple, so <math>DF = \frac{80}{25} = \frac{16}{5}</math>. | ||
+ | |||
+ | <math>16 + 5 = m + n = \boxed{\textbf{(B) }21}</math>. | ||
+ | |||
+ | ~JT0543164 | ||
+ | |||
+ | ==Video Solution by Pi Academy== | ||
+ | |||
+ | https://youtu.be/2wrOFzxYCcM?si=J-u-_5Yb8o3hXrAp | ||
+ | |||
+ | ~ Pi Academy | ||
+ | |||
+ | ==Video Solution 2== | ||
+ | |||
+ | https://www.youtube.com/watch?v=X0YJfFiBy0g | ||
+ | |||
+ | https://youtu.be/XZBKnobK-JU?t=3064 | ||
== See also == | == See also == | ||
{{AMC12 box|year=2013|ab=B|num-b=18|num-a=20}} | {{AMC12 box|year=2013|ab=B|num-b=18|num-a=20}} | ||
− | + | {{AMC10 box|year=2013|ab=B|num-b=22|num-a=24}} | |
− | |||
{{MAA Notice}} | {{MAA Notice}} |
Latest revision as of 15:00, 29 May 2025
- The following problem is from both the 2013 AMC 12B #19 and 2013 AMC 10B #23, so both problems redirect to this page.
Contents
[hide]Problem
In triangle ,
,
, and
. Distinct points
,
, and
lie on segments
,
, and
, respectively, such that
,
, and
. The length of segment
can be written as
, where
and
are relatively prime positive integers. What is
?
Diagram
Solution 1
Since , quadrilateral
is cyclic. It follows that
, so
are similar. In addition,
. We can easily find
,
, and
using Pythagorean triples.
So, the ratio of the longer leg to the hypotenuse of all three similar triangles is , and the ratio of the shorter leg to the hypotenuse is
. It follows that
.
Let . By Ptolemy's Theorem, we have
Dividing by
we get
so our answer is
.
~Edits by BakedPotato66
Solution 2
From solution 1, we know that and
. Since
, we can figure out that
. We also know what
is so we can figure what
is:
. Quadrilateral
is cyclic, implying that
= 180°. Therefore,
, and triangles
. Solving the resulting proportion gives
. Therefore,
.
and our answer is
.
because of
.
. Lets say
. So
and
so
Solution 3
Let and
Noting that
is the altitude from
to
and
Notice that since
is perpendicular to
,
is similar to
Therefore since
is a
-
-
triangle,
Applying Law of Cosines on
we have,
Next notice that
Now we apply Law of Cosines on
Finally using Pythagorean Theorem on
we have,
Solving for the nonnegative value for
we find
Solution 4
If we draw a diagram as given, but then add point on
such that
in order to use the Pythagorean theorem, we end up with similar triangles
and
. Thus,
and
, where
is the length of
. Using the Pythagorean theorem, we now get
and
can be found out noting that
is just
through base times height (since
, similar triangles gives
), and that
is just
. From there,
Now,
, and squaring and adding both sides and subtracting a 169 from both sides gives
, so
. Thus, the answer is
.
Solution 5 (Power of a Point)
First, we find ,
, and
via the Pythagorean Theorem or by using similar triangles. Next, because
is an altitude of triangle
,
. Using that, we can use the Pythagorean Theorem and similar triangles to find
and
.
Points ,
,
, and
all lie on a circle whose diameter is
. Let the point where the circle intersects
be
. Using power of a point, we can write the following equation to solve for
:
Using that, we can find
, and using
, we can find that
.
We can use power of a point again to solve for :
Thus,
.
Solution 6 (Coord Bash)
If we draw the diagram like above, but make the base, we can set
and
, as
can quickly be verified to be
by Pythagorean triples or similar triangles. Construct
on
such that
. This implies
as
, and
. Also construct
such that
.
Line has a slope of
by slope formula. Since
and
, the equation of
.
Furthermore, can now be expressed as
Since we know
we can solve for
with the perpendicular slope formula like so:
Plugging into
, we get
Since
, we get that
has side lengths of
and .
Clearly, is a
pythagorean triple, so
.
.
~JT0543164
Video Solution by Pi Academy
https://youtu.be/2wrOFzxYCcM?si=J-u-_5Yb8o3hXrAp
~ Pi Academy
Video Solution 2
https://www.youtube.com/watch?v=X0YJfFiBy0g
https://youtu.be/XZBKnobK-JU?t=3064
See also
2013 AMC 12B (Problems • Answer Key • Resources) | |
Preceded by Problem 18 |
Followed by Problem 20 |
1 • 2 • 3 • 4 • 5 • 6 • 7 • 8 • 9 • 10 • 11 • 12 • 13 • 14 • 15 • 16 • 17 • 18 • 19 • 20 • 21 • 22 • 23 • 24 • 25 | |
All AMC 12 Problems and Solutions |
2013 AMC 10B (Problems • Answer Key • Resources) | ||
Preceded by Problem 22 |
Followed by Problem 24 | |
1 • 2 • 3 • 4 • 5 • 6 • 7 • 8 • 9 • 10 • 11 • 12 • 13 • 14 • 15 • 16 • 17 • 18 • 19 • 20 • 21 • 22 • 23 • 24 • 25 | ||
All AMC 10 Problems and Solutions |
These problems are copyrighted © by the Mathematical Association of America, as part of the American Mathematics Competitions.